Prove 2coth^(-1)(e^x) = ln[cosech x + coth x]

  • Thread starter Thread starter DryRun
  • Start date Start date
Click For Summary
To prove the equation 2coth^(-1)(e^x) = ln[cosech x + coth x], the initial approach involves manipulating the left-hand side (LHS) by expressing it in terms of tanh^(-1). The transformation leads to ln[(1+(e^-x))/(1-(e^-x))], but progress stalls at this point. A suggestion is made to work from the right-hand side (RHS) and use the substitution u = e^x to simplify the process. This substitution is expected to clarify the relationship between the two sides of the equation. The discussion emphasizes the importance of strategic manipulation in proving the identity.
DryRun
Gold Member
Messages
837
Reaction score
4
Homework Statement
Prove 2coth^(-1)(e^x) = ln[cosech x + coth x]

The attempt at a solution
I'm starting with the L.H.S. first, which is conventional, i think.

2coth^(-1)(e^x)

= 2tanh^(-1)(e^-x)

= 2[1/2 ln [(1+(e^-x))/(1-(e^-x))]

= ln [(1+(e^-x))/(1-(e^-x))]

I'm stuck.

So, i tried to cheat a little, by expanding the R.H.S. and see if i could find a way to convert the last step above into the R.H.S. but my copybook has become something of a painting.
 
Physics news on Phys.org
sharks said:
Homework Statement
Prove 2coth^(-1)(e^x) = ln[cosech x + coth x]

The attempt at a solution
I'm starting with the L.H.S. first, which is conventional, i think.

2coth^(-1)(e^x)

= 2tanh^(-1)(e^-x)

= 2[1/2 ln [(1+(e^-x))/(1-(e^-x))]

= ln [(1+(e^-x))/(1-(e^-x))]

I'm stuck.

So, i tried to cheat a little, by expanding the R.H.S. and see if i could find a way to convert the last step above into the R.H.S. but my copybook has become something of a painting.

Hints: work from the RHS, and substitute u = e^x. Will make everything much clearer.
 
Question: A clock's minute hand has length 4 and its hour hand has length 3. What is the distance between the tips at the moment when it is increasing most rapidly?(Putnam Exam Question) Answer: Making assumption that both the hands moves at constant angular velocities, the answer is ## \sqrt{7} .## But don't you think this assumption is somewhat doubtful and wrong?

Similar threads

  • · Replies 14 ·
Replies
14
Views
2K
  • · Replies 5 ·
Replies
5
Views
1K
Replies
5
Views
1K
Replies
7
Views
2K
Replies
2
Views
1K
Replies
11
Views
2K
  • · Replies 4 ·
Replies
4
Views
2K
  • · Replies 3 ·
Replies
3
Views
1K
  • · Replies 5 ·
Replies
5
Views
2K
Replies
2
Views
1K